What is Angular momentum: Definition and 1000 Discussions

In physics, angular momentum (rarely, moment of momentum or rotational momentum) is the rotational equivalent of linear momentum. It is an important quantity in physics because it is a conserved quantity—the total angular momentum of a closed system remains constant.
In three dimensions, the angular momentum for a point particle is a pseudovector r × p, the cross product of the particle's position vector r (relative to some origin) and its momentum vector; the latter is p = mv in Newtonian mechanics. Unlike momentum, angular momentum depends on where the origin is chosen, since the particle's position is measured from it.
Just as for angular velocity, there are two special types of angular momentum of an object: the spin angular momentum is the angular momentum about the object's centre of mass, while the orbital angular momentum is the angular momentum about a chosen center of rotation. The total angular momentum is the sum of the spin and orbital angular momenta. The orbital angular momentum vector of a point particle is always parallel and directly proportional to its orbital angular velocity vector ω, where the constant of proportionality depends on both the mass of the particle and its distance from origin. The spin angular momentum vector of a rigid body is proportional but not always parallel to the spin angular velocity vector Ω, making the constant of proportionality a second-rank tensor rather than a scalar.
Angular momentum is an extensive quantity; i.e. the total angular momentum of any composite system is the sum of the angular momenta of its constituent parts. For a continuous rigid body or a fluid the total angular momentum is the volume integral of angular momentum density (i.e. angular momentum per unit volume in the limit as volume shrinks to zero) over the entire body.
Torque can be defined as the rate of change of angular momentum, analogous to force. The net external torque on any system is always equal to the total torque on the system; in other words, the sum of all internal torques of any system is always 0 (this is the rotational analogue of Newton's Third Law). Therefore, for a closed system (where there is no net external torque), the total torque on the system must be 0, which means that the total angular momentum of the system is constant. The conservation of angular momentum helps explain many observed phenomena, for example the increase in rotational speed of a spinning figure skater as the skater's arms are contracted, the high rotational rates of neutron stars, the Coriolis effect, and the precession of gyroscopes. In general, conservation limits the possible motion of a system but does not uniquely determine it.
In quantum mechanics, angular momentum (like other quantities) is expressed as an operator, and its one-dimensional projections have quantized eigenvalues. Angular momentum is subject to the Heisenberg uncertainty principle, implying that at any time, only one projection (also called "component") can be measured with definite precision; the other two then remain uncertain. Because of this, the axis of rotation of a quantum particle is undefined. Quantum particles do possess a type of non-orbital angular momentum called "spin", but this angular momentum does not correspond to a spinning motion.

View More On Wikipedia.org
  1. Twigg

    I Does putting a hydrogen atom in a box mix angular momentum states?

    If you put a hydrogen atom in a box (##\psi=0## on the walls of the box), spherical symmetry will be broken so ##n##,##l##,##m_l## are no longer guaranteed to be good quantum numbers. In general, the new solutions will be a linear combination of all the ##|n,l,m_l\rangle## states. I know that...
  2. Z

    What happens to a ball placed on a moving conveyor belt?

    Here is my depiction of the initial state: Note that the presence of ##f_k## means the ball is initially slipping. We also know that the linear and angular speeds of the ball are increasing in time. At some point, the ball should stop slipping. The condition for no slipping is that the speed...
  3. Rikudo

    Integration in angular momentum

    https://www.physicsforums.com/threa...f-a-translating-and-rotating-pancake.1005990/ So,I think I posted this in the wrong place. So, I will move it to here. Here, in post #6, it is stated that ##\int R dm = M R##. As far as I know, R change from time to time and it is not constant. Hence, isn't...
  4. Rikudo

    Confusion in choosing an origin point for angular momentum

    I am currently reading David Morin book and found this statement : ##\,\,\,\,\,\,\,\,## "It is important to remember that you are free to choose your origin from the legal possibilities of fixed points or the CM" Is it really alright to choose the center of a...
  5. LCSphysicist

    What happens to an atom's angular momentum when it absorbs an electron?

    I was thinking a little about how the absorption of angular momentum occurs from the point of view of QM. For example, suppose we have an atom A and an electron $e^-$. The electron $e^-$ is ejected from a source radially in direction of the center of the atom. Suppose that the atom has net...
  6. R

    Angular momentum of a particles in the form of ##L = mr^2\omega##

    ##\vec{L} = \vec{P} \times\vec{r}## ##L = mvr sin \phi##, where P = mv Since ##\vec{r}## and ##\vec{v}## are always perpendicular, ##\phi## = 90. Then, ##L = mvr## At this point, I don't see how to get ##L = mvr = mr^2\omega##, using ##\omega = \dot{\phi}## I know that ##\omega =...
  7. J

    I Do measurements modify angular momentum and energy?

    While physics is generally believed to be CPT symmetric, there are processes for which such symmetry is being questioned - especially the measurement. One of examples of (allegedly?) going out of QM unitary evolution is atom deexcitation - we can save its reversibility by remembering about...
  8. Rikudo

    I Total angular momentum of a translating and rotating pancake

    I have read Classical Mechanics book by David Morin, and there are some parts that I do not understand from its derivation. Note : ## V## and ##v## is respectively the velocity of CM and a particle of the body relative to the fixed origin , while ##v'## is velocity of the particle relative to...
  9. A

    I Time derivative of the angular momentum as a cross product

    I am trying to find the equations of motion of the angular momentum ##\boldsymbol L## for a system consisting of a particle of mass ##m## and magnetic moment ##\boldsymbol{\mu} \equiv \gamma \boldsymbol{L}## in a magnetic field ##\boldsymbol B##. The Hamiltonian of the system is therefore...
  10. M

    I Derivation of an angular momentum expression

    Hello! I found this formula in several places for the total angular momentum of a particle with intrinsic spin 1/2 and angular momentum l=1 in the non-relativistic limit: $$\frac{1}{\sqrt{4 \pi}}(-\sigma r /r )\chi$$ where ##\sigma## are the Pauli matrices and ##\chi## is the spinor. Can someone...
  11. A.T.

    B Falling Cat - Rotation with Zero Total Angular Momentum

    I made a new version of the falling cat video, with narration. It explains how cats turn around while having zero net angular momentum during the fall:
  12. K

    I Rotation about two axes and angular momentum

    I've a body having initial angular velocity at ## t=0 ## as shown. The axis shown are fixed in inertial space and initially match with the principal axis. I want to find the infinitesimal change at ##t+\Delta t## in the angular momentum along the ##z## axis. I've seen the following approach...
  13. K

    I How Does the Book's Formula for Angular Momentum Differ from Mine?

    A disc initially has angular velocities as shown It's angular momentum along the y-axis initially is ##L_s## I tried to find its angular momentum and ended up with this:##L=I_{x} \omega_{x}+I_{y} w_{y}+I_{z} z_{z}##The z component of angular momentum is thus ##L_{z}=I_{z} \omega_{z}## However...
  14. Viona

    B The average value of S operator

    While reading in the book of Introduction to Quantum Mechanics by David Griffith in the section of Fine structure of Hydrogen: spin- orbit coupling, he said that the average value of S operator is considered to be the projection of S onto J. I could not understand why he assumed that. please...
  15. Homestar1

    B Electron angular momentum, gyroscope?

    Any spinning item, proton, electron, even planet, has angular momentum that creates force. How can an electron exist in a random orbital cloud around a spinning proton if it has an angular momentum and requires force to alter from any circular orbital plane (like a planet orbiting a star)?
  16. PiEpsilon

    Analyzing an Angular Impulse Problem

    What we know: The ball is dropped at the tip A with some speed ##v_0## and rebounds with speed ##v##. This collision produces an angular impulse, changing the angular momentum of the bar with the flywheels. Solution inspired by an answer provided by @TSny in the similar question. Angular...
  17. PiEpsilon

    Elastic collision of particle and rotating disc

    Consider the system of the mass and uniform disc. Since no external forces act on the system, the angular momentum will be conserved. For elastic collision, the kinetic energy of the system stays constant.Measuring angular momentum from the hinge: ##\vec L_i = Rmv_0 \space\hat i + I \omega_0...
  18. I

    Ballentine Problem 7.1 Orbital Angular Momentum

    Find the probability distributions of the orbital angular momentum variables ##L^{2}## and ##L_{z}## for the following orbital state functions: ##\Psi(x) = f(r) sin(\theta) cos(\theta)## ##\Psi(x) = f(r) cos^{2}(\theta)##I am aware that the prob. distribution of an observable is ##|<a_{n} |...
  19. E

    I Finding the angular momentum of a Kerr black hole

    [Mentor Note -- Specialized question moved to the general technical forums] Homework Statement:: To show that ##J = Ma## for the charged Kerr metric [Wald Ch. 11 Pr. 6] Relevant Equations:: \begin{align*} \mathrm{d}s^2 = &- \left( \frac{\Delta - a^2 \sin^2{\theta}}{\Sigma}\right) \mathrm{d}t^2...
  20. akashpandey

    Direction of Angular velocity and Angular momentum?

    I am very confused when textbooks say the direction of Angular velocity is perpendicular ot radius and theta for that matter direction is in perpendicular direction. I know this comes from cross product rule but what is the meaning of Angular velocity and Angular momentum directing in upward...
  21. M

    The propagator of eigenstates of the Total Angular Momentum

    To show that when ##[J^2, H]=0 ## the propagator vanishes unless ##j_1 = j_2## , I did (##\hbar =1##) $$ K(j_1, m_1, j_2 m_2; t) = [jm, e^{-iHt}]= e^{iHt} (e^{iHt} jm e^{-iHt}) - e^{-iHt} jm $$ $$ = e^{iHt}[jm_H - jm] $$ So we have $$ \langle j_1 m_1 | [jm, e^{-iHt} ] | j_2 m_2 \rangle $$ $$ =...
  22. H

    What is the angular momentum of the clay-rod system?

    I calculate in this way : Angular Momentum = I W = [ ( 1/12 ML^2 + m(L/2)^2 ] (V/ L/2) = [ 1/12 ML^2 + 1/4 mL^2 ] 2V/L = 2VL/4 [ M/3 + M] but can not find a matching answer. Why?
  23. hquang001

    Rotational motion and angular momentum

    mball = 2 kg, mputty = 0.05 kg, L = 0.5 m, v = 3m/s a) Moment of inertia : I = (2mball + mputty ). ¼ L^2 = 0.253125 kg.m^2 Linitial = Lfinal => mputty. v. r = I.ω => ω = (4.mputty.v.r) / I = 0.148 rad/s b) K initial = 1/2 m v^2 = 0.225 J K final = 1/2 Iω^2 = 2.85.10^(-3) J => Kfinal /...
  24. L

    I Angular Momentum Hydrogen Atom Problem: Physically Explained When L=0

    In quantum mechanics hydrogen atom problem ##L=\sqrt{l(l+1)}\hbar##. What that means physically when ##L=0##?
  25. M

    I Changes in angular momentum for ro-vibrational transitions

    Hello! If we have a transition between 2 ro-vibrational levels of the same electronic state of a diatomic molecule the selection rules require for the changes in the rotational quantum number J that ##\Delta J = \pm 1##. Why can't we have ##\Delta J = 0##? The photon carries one unit of angular...
  26. K

    I Electron angular momentum in diatomic molecules

    Hello! I just started reading some molecular physics and I am a bit confused about the electron angular momentum in diatomic molecules. Let's say we have just 2 protons and an electron for simplicity and we are in the Born-Oppenheimer approximation, so we assume that the nuclei are fixed in...
  27. A

    Exploring Angular Momentum: Examining Earth & Bike Wheels

    Take for example earth. Earth has angular momentum about its own axis. However, if we ignore the orbital portion, the angular momentum of the Earth relative to the sun's axis is the same. Another example is the spinning bike wheel/person holding it in a chair. It has angular momentum about its...
  28. C

    Solving for $$\omega_2$$ using Conservation of Angular Momentum

    Unfortunately, I couldn't arrive to the correct answer ($$=0.28mL^2 \omega^2$$ ) and will be happy to understand what am I doing wrong. **My attempt:** Using $$ E_k = \frac{1}{2} I \omega^2 $$ I obtain that the difference I need to calculate is $$ \frac{1}{2} (2mL^2)(0.8\omega)^2 +...
  29. A

    Spinning Bike Wheel Example, how is angular momentum conserved?

    In the classic example of a person holding a spinning bike wheel, as they flip the wheel over, angular momentum is conserved by the person/chair spinning with 2x the angular momentum of the initial wheel. Not questioning that. However, I thought ang momentum is always conserved about a...
  30. Kaguro

    Angular momentum of orbit from orbit parameters and mass of sun

    L = mvr = mr (dr/dt) = 2m*r*(dr/dt)/2 = 2m*(dA/dt) So, A = (L/2m)T so, ## L = \frac{2 \pi a b m}{T}## Now, ##T^2 = \frac{4 \pi^2}{GM} a^3## So from all these, I get ##L = \sqrt{ \frac{GM m^2 b^2}{a}}## But answer given is ##L = \sqrt{ \frac{2GM m^2 ab}{a+b}}## (This, they have derived from...
  31. Kaguro

    Can spin angular momentum get converted to orbital angular momentum?

    I know that in QM, there is LS coupling. So the interaction is there. But is such an interaction possible in macroscopic objects like a planet?
  32. Leo Liu

    Why is angular momentum measured at a non inertial CoM conserved?

    Question 7.6 Official solution It seems that the solution uses the conservation of angular momentum to solve this question (τ=0). But the problem is that the frame is set on the centre of mass of the guy, which is non inertial. I would like to know why it is correct to do it this way. My...
  33. Ayandas1246

    Conceptual questions about Angular Momentum Conservation and torque

    List of relevant equations: Angular Momentum = L (vector) = r(vector) x p(vector) Angular velocity of rotating object = w(vector), direction found using right hand rule. Torque = T(vector) = dL(vector)/dt I have a few questions about torque and angular momentum direction and...
  34. Leo Liu

    Is angular momentum conserved in a non-inertial frame?

    Question: If we place the frame of reference on an accelerating point, does the total rotational momentum still remain the same? I attempted to solve this question by manipulating the equations as shown below. $$\text{Define that }\vec r_i=\vec R+\vec r_i'\text{, where r is the position vector...
  35. J

    Angular Momentum: Spinning Mass

    First I calculated the momentum of m1. Since m2 was at rest after the collision, all its momentum was transferred, so m1 has a momentum of 158 i hat. L=r x p, so its 916 k hat. This would also be the change in L because it was initially 0 when m1 had no velocity, so I know this is the net...
  36. J

    Angular Momentum Problem: Rotation Rate

    First I found the moment of inertia, I=1.8(5.5^2+3.9^2+4.9^2) =125.046 Then I tried to find the rotation rate using the equation L=rotation rate*I rotation rate=3773/125.046=30.173 But the answer is suppose to be 21.263?
  37. J

    Angular Momentum Problem: Torque after fall

    I know how to get to the answer but that's what is confusing me. To find final velocity I multiply the acceleration by the time the object fell. Then multiply the velocity by the mass to get momentum. Now the angular momentum is r x p. Since the initial angular momentum was 0, this was also...
  38. J

    Angular Momentum Problem: Determining Angular Acceleration

    So I first tried to find L using torque, Torque=d/dt*L, and took the integral of this. Ended up with 23.28484t Now I square the equation L=rotation rate*I to get L^2=rotation acceleration *I^2 Angular acceleration=L^2/I^2 I feel like I am doing something wrong though, this doesn't give the...
  39. M

    Conservation of Angular Momentum -- Child jumping onto a Merry-Go-Round

    So we know that the initial intertia of the merry go round is 250 kg m^2 and its angular speed is 10 rpm. MGRs angular momentum would be L=Iw=250(10)=2500kg m^2 rpm. We know the mass if the child is 25kg, and the child's linear velocity is 6m/s. We convert linear to angular w= v/r = 6/2 =...
  40. WonderKitten

    Conservation of angular momentum

    Hi, I have the following problem: A homogeneous disc with M = 1.78 kg and R = 0.547 m is lying down at rest on a perfectly polished surface. The disc is kept in place by an axis O although it can turn freely around it. A particle with m = 0.311 kg and v = 103 m/s, normal to the disc's surface at...
  41. Saptarshi Sarkar

    Conservation of angular momentum under central forces

    I know that the force must be a central force and that under central forces, angular momentum is conserved. But I am unable to mathematically show if the angular and linear momentum are constants. Radial Momentum ##p=m\dot r = ma\dot \theta=ma\omega## Angular Momentum ##L=mr^2\dot\theta =...
  42. JD_PM

    Dirac-Hamiltonian, Angular Momentum commutator

    We want to show that ##[\hat{ \vec H}, \hat{ \vec L}_T]=0##. I made a guess: we know that ##[\hat{ \vec H}, \hat{ \vec L}_T]=[\hat{ \vec H}, \hat{ \vec L}] + \frac 1 2 [\hat{ \vec H}, \vec \sigma]=0## must hold. I have already shown that $$[\hat{ \vec H}, -i \vec r \times \vec \nabla]= -...
  43. B

    B Does Jupiter have an angular momentum problem?

    I'm sure I've read somewhere that Jupiter has 99% of the solar system's angular momentum, which shouldn't be the case. However, I can't find a source for this, and any search online for the topic doesn't bring up any science sites. Did I mis-remember?
  44. T

    Clarification on angular momentum

    I am confused with the following two questions: 1. A particle moves under the influence of a central force directed toward a fixed origin ##O##. Explain why the particle's angular momentum about ##O## is constant. 2. Consider a planet orbiting the fixed sun. Take the plane of the planet's...
  45. E

    What are angular momentum and torque?

    Wikipedia says that they are the equivalents of momentum and force in rotational motion but I don't understand why this comparison is possible. The torque's dimension is N*m it seems like energy. What is this energy? Why angular momentum is not mass times angular velocity?
  46. A

    Angular momentum conserved with multiple gears?

    Say we have a motor attached to the Earth with gear A, that drives identically sized gear B. Gear B spins on its own axis and but is also attached to ground. Torque between gears is equal. Technically each gear has equal but opposite AM right, but If I take Earth into account, how is Ang...
  47. E

    How Is Angular Momentum Calculated Without Considering Gravity and Viscosity?

    we neglect gravity and viscosity efects i really can't understand how does the author managed to get the equation in the image
  48. jjson775

    Understanding Energy Conservation in Quantized Angular Momentum Transitions

    I don’t understand how energy is conserved here. The energy of the atom when n=5 is -.544eV. The energy of the photon is 1.14eV. After release, the energy of the atom is -.544 - 1.14 = 1.68eV. Using this value, I get n = 2.67, not an integer, so n = 3 and the atom has energy = -1.51 eV. I...
Back
Top